What Are the Different Coordinate Systems Used in Calculus?

  • Thread starter Thread starter JasonJo
  • Start date Start date
  • Tags Tags
    Calc 3 Coordinate
Click For Summary
The discussion focuses on converting expressions between different coordinate systems in calculus, specifically polar, cylindrical, and Cartesian coordinates. Users seek assistance in rewriting second derivatives in polar coordinates and expressing surfaces in cylindrical coordinates, highlighting confusion around Cartesian coordinates. The conversation emphasizes the need for a clear method to transition between these systems, particularly for those struggling with the concepts. Participants share their approaches to derivatives and coordinate transformations, aiming to clarify the geometric interpretations involved. Overall, the thread serves as a collaborative effort to enhance understanding of coordinate systems in calculus.
JasonJo
Messages
425
Reaction score
2
1) Rewrite the following expression in polar coordinates:
(second derivative of z with respect to x) + (second derivative of z with respect to y)

where x=rcos(theta)
y = rsin(theta)

i had first derivative of z with respect to x = (dz/dx)(dx/dr) + (dz/dx)(dx/d(theta))

same concept for y

and then i just took the derivative of my dz/dz and dz/dy variables again

2) describe this surface in cylindrical coordinates
z = rcos3(phi)

express in cartesian coordinates

i really do not get cartesian coordinates

can anyone show me the generic method?

3) x = psin(phi)cos(theta)
y = psin(phi)sin(theta)
z = pcos(phi)

near which points of R^3 can we solve for: p phi and theta in terms of x, y and z, describe the geometry behind your answer.

again, i am really really weak in coordinate systems.

any help is appreciated, thanks guys
 
Physics news on Phys.org
what does z = in #1

x = p sin phi cos theta

y =p sin phi sin theta

z = p cos phi
 
Question: A clock's minute hand has length 4 and its hour hand has length 3. What is the distance between the tips at the moment when it is increasing most rapidly?(Putnam Exam Question) Answer: Making assumption that both the hands moves at constant angular velocities, the answer is ## \sqrt{7} .## But don't you think this assumption is somewhat doubtful and wrong?

Similar threads

  • · Replies 4 ·
Replies
4
Views
2K
  • · Replies 4 ·
Replies
4
Views
2K
  • · Replies 7 ·
Replies
7
Views
1K
  • · Replies 2 ·
Replies
2
Views
2K
  • · Replies 4 ·
Replies
4
Views
2K
  • · Replies 1 ·
Replies
1
Views
2K
  • · Replies 8 ·
Replies
8
Views
6K
  • · Replies 1 ·
Replies
1
Views
2K
  • · Replies 3 ·
Replies
3
Views
3K
Replies
6
Views
2K